LSAT and Law School Admissions Forum

Get expert LSAT preparation and law school admissions advice from PowerScore Test Preparation.

 Administrator
PowerScore Staff
  • PowerScore Staff
  • Posts: 8937
  • Joined: Feb 02, 2011
|
#90593
Complete Question Explanation

Must Be True. The correct answer choice is (A).

Answer choice (A): This is the correct answer choice.

Answer choice (B):

Answer choice (C):

Answer choice (D):

Answer choice (E):

This explanation is still in progress. Please post any questions below!
 KindaConfused
  • Posts: 2
  • Joined: Aug 13, 2021
|
#91906
Hi, can someone provide an explanation for this one?
 Adam Tyson
PowerScore Staff
  • PowerScore Staff
  • Posts: 5271
  • Joined: Apr 14, 2011
|
#91950
Happy to help, Kinda! I'll keep it simple here, but we expect to have a more complete official explanation soon.

The premises are about people making economically motivated decisions and whether that explains popular support for political parties. Some sociologists say it cannot, on the grounds that "a simple phenomenon" can't explain a complex one, and the success of a political party is a complex phenomenon.

There's a big gap in the argument: The sociologists talk about simple phenomena, but never talk about those economically-motivated decisions that are under discussion. To close that gap, those sociologists must believe that those decisions are, in fact, a simple phenomenon! There's the prephrase: connect those two ideas in the correct answer, and reject any answers that fail to close that gap. We call that the Supporter approach to Assumption questions (and while this question stem reads like a Must Be True question, the emphasis on what those sociologists "believe" leads me to treat it like an Assumption question, especially because there is an argument in the stimulus rather than just a fact set).

Answer A makes that connection for us and is therefore correct. They muddied the waters a bit by saying that those decision need not be complex, instead of just saying they are simple, but if they have to believe they are simple then they have to believe they are not complex, so it works. If you apply the Negation Technique to that answer it will tell us that those economic decisions must be a complex phenomenon, which would ruin the argument made by the sociologists, which proves that they must have believed in that answer.
User avatar
 supernerd
  • Posts: 7
  • Joined: Nov 12, 2021
|
#92054
why not B?
 Adam Tyson
PowerScore Staff
  • PowerScore Staff
  • Posts: 5271
  • Joined: Apr 14, 2011
|
#92059
The short answer, supernerd, is "because the sociologists don't have to believe that." To prove that they don't have to believe that, imagine what would happen to their argument if they didn't believe that. What if they don't believe that there are many complex causes, but just one complex cause? In that case, their argument would still work, as long as those economic decisions people make are simple rather than complex. They have to believe that there is SOME complex phenomenon involved, but they don't have to believe there are MANY such phenomena causing things to happen.
User avatar
 Trying My Best
  • Posts: 6
  • Joined: Jan 05, 2022
|
#93202
Adam Tyson wrote: Tue Nov 09, 2021 2:57 pm
There's a big gap in the argument: The sociologists talk about simple phenomena, but never talk about those economically-motivated decisions that are under discussion. To close that gap, those sociologists must believe that those decisions are, in fact, a simple phenomenon! There's the prephrase: connect those two ideas in the correct answer, and reject any answers that fail to close that gap. We call that the Supporter approach to Assumption questions (and while this question stem reads like a Must Be True question, the emphasis on what those sociologists "believe" leads me to treat it like an Assumption question, especially because there is an argument in the stimulus rather than just a fact set).
My question from this explanation is that I did not catch on to the idea to treat this question as an Assumption question rather than a must be true. Therefore, I felt that letter B was a supported inference that I could draw based on the stimulus. Although, now as I read this thread, I realized "many" should have been a red flag. I feel like this question's efficiency for attack is heavily reliant on being able to close that gap that we so often see in both Assumption and Justify questions. Should we always be cognizant of closing gaps in argumentative must be true questions?
 Adam Tyson
PowerScore Staff
  • PowerScore Staff
  • Posts: 5271
  • Joined: Apr 14, 2011
|
#93205
You bet, Trying! When a gap is present, you must always be aware of it, because the presence of that gap could be the key to prephrasing the correct answer whether the question is an Assumption, Strengthen, Justify, or even Weaken or Flaw. And while it is rare to find an argument, and thus a gap, in a Must Be True question, if there is such a gap then it would still be a reasonable place to look to determine what must be true!
User avatar
 Lsatbeast69
  • Posts: 4
  • Joined: Sep 05, 2022
|
#97613
Is D Wrong because it is too strong to say "never complex"
User avatar
 atierney
PowerScore Staff
  • PowerScore Staff
  • Posts: 215
  • Joined: Jul 06, 2021
|
#97632
D is wrong because the "popular support" is the complex phenomenon itself, according to the sociologists. Thus, it's the opposite of the right answer.
User avatar
 mkloo11
  • Posts: 20
  • Joined: Mar 30, 2023
|
#100800
Hi! Sorry to reopen this, but wouldn't someone be willing to explain how "need not" in A works? I've never seen that phrase used in this way. It made me pause, and while in this case it wasn't dubious enough to dissuade me from choosing that answer choice, I'm still not sure how to use the Negation Technique with that language. Thank you!

Get the most out of your LSAT Prep Plus subscription.

Analyze and track your performance with our Testing and Analytics Package.